summarize the steps a scientist might take to determine if a new drug Works in cancer patients

blah help​

Answers

Answer 1

1. Identification of the problem whether the new drug works in cancer patients.

2. Create a hypothesis like, if the new drugs works on all types of cancers.

3. Variables like placebo effect of the drug and its dosages to be administered.

4. Creating experiments to test the viability of the drug.

5. Analyzing results of the experimentation.

6. Form a conclusion and test further depending on the result of the experiments.

I hope this answer helps you out. Brainliest would be appreciated :)

Related Questions

A circular loop of wire of area 25 cm2 lies in the plane of the paper. A decreasing magnetic field B is coming out of the paper. What is the direction of the induced current in the loop?

Answers

Answer:

counterclockwise

Explanation:

given data

area = 25 cm²

solution

We know that a changing magnetic field induces the current and induced emf is express as

[tex]\epsilon = -N \frac{d \phi }{dt}[/tex]     ..................................1

and we will get here direction of the induced current in the loop that is express by the Lens law that state that the direction of induces current is such that the magnetic flux due to the induced current opposes the change in magnetic flux due to the change in magnetic field

so when magnetic field decrease and point coming out of the paper.

so induced current in the loop will be counterclockwise

Choose only one correct option. Explanation needed.

Answers

Answer:

[tex]\large \boxed{\mathrm{C. \ \ \frac{500}{7 \times 15 \times 8} \ g/cm^3 }}[/tex]

Explanation:

[tex]\displaystyle \sf Density = \frac{mass}{volume}[/tex]

[tex]\displaystyle \rho = \frac{m}{V}[/tex]

[tex]\sf mass=500 \ g[/tex]

[tex]\sf volume \ of \ a \ cuboid=width \times length \times height=( 7 \times 15 \times 8) \ cm^3[/tex]

[tex]\displaystyle \rho = \frac{500}{7 \times 15 \times 8}[/tex]

what is the value of x if x-36=5?​

Answers

Answer:

Therefore, the value of x is 41

Explanation:

x=5+36

x=41

g Can a rigid body experience any ACCELERATION when the resultant force acting on that rigid body is zero? Explain.Can a rigid body experience any ACCELERATION when the resultant force acting on that rigid body is zero? Explain.

Answers

Answer:

No, a rigid body cannot experience any acceleration when the resultant force acting on the body is zero.

Explanation:

If the net force on a body is zero, then it means that all the forces acting on the body are balanced and cancel out one another. This sate of equilibrium can be static equilibrium (like that of a rigid body), or dynamic equilibrium (that of a car moving with constant velocity)

For a body under this type of equilibrium,

ΣF = 0   ...1

where ΣF is the resultant force (total effective force due to all the forces acting on the body)

For a body to accelerate, there must be a force acting on it. The acceleration of a body is proportional to the force applied, for a constant mass of the body. The relationship between the net force and mass is given as

ΣF = ma   ...2

where m is the mass of the body

a is the acceleration of the body

Substituting equation 2 into equation 1, we have

0 = ma

therefore,

a = 0

this means that if the resultant force acting on a rigid body is zero, then there won't be any force available to produce acceleration on the body.

Question 18(Multiple Choice Worth 2 polnis)
When riding your skateboard you crash into a curb, the skateboard stops, and you continue moving forward. Which law of
motion is being described in this scenario?
O Law of Universal Gravitation
o Newton's Second Law of Motion
o Law of Conservation of Energy
o Newton's First Law of Motion​

Answers

Last point, Newton’s first law of motion is the correct answer.

Hope this helps ya

Calculate the magnetic field and its direction at point P, which is 2.0 cm away from the top wire and 4.0 cm from the bottom wire. Assume both wires are infinitely long and each carries a current of 1.5 A.

Answers

Answer:

Explanation :  B = μ₀i / 2πr

B1 = (2×10^-7) × 1.5/0.02

       = 15 μT

B2 = (2×10^-7 ) ×  1.5/0.04

       = 7.5 μT

total field = 15 μT + 7.5 μT

                  = 22.5 μT

An astronomer is measuring the electromagnetic radiation emitted by two stars, both of which are assumed to be perfect blackbody emitters. For each star she makes a plot of the radiation intensity per unit wavelength as a function of wavelength. She notices that the curve for star A has a maximum that occurs at a shorter wavelength than does the curve for star B. What can she conclude about the surface temperatures of the two stars

Answers

Answer:

Star A has a higher surface temperature than star B.

Explanation:

The effective temperature of a star can be determined by means of its spectrum and Wien's displacement law:

[tex]T = \frac{2.898x10^{-3} m. K}{\lambda max}[/tex] (1)

Where T is the effective temperature of the star and [tex]\lambda_{max}[/tex] is the maximum peak of emission.  

A body that is hot enough emits light as a consequence of its temperature. For example, if an iron bar is put in contact with fire, it will start to change colors as the temperature increase, until it gets to a blue color, that scenario is known as Wien's displacement law. Which establishes that the peak of emission for the spectrum will be displaced to shorter wavelengths as the temperature increase and higher wavelengths as the temperature decreases.

Therefore, star A has a higher surface temperature than star B, as it is shown in equation 1 since T and [tex]\lambda max[/tex] are inversely proportional.

Dharna is said to be concentration it is true or false​

Answers

dharna is said to be concentration.. it is true

A 10 n force is applied horizontally on a box to move it 10 m across a frictionless surface. How much work was done to move the box?

Answers

Given from question
Force = 10 N
Displacement = 10 m
Work done = ?
We know that
Work done = force X displacement
So 10 X 10
100
Work done = 100J answer

Answer:

[tex]\boxed {\boxed {\sf 100 \ J}}[/tex]

Explanation:

We are asked to calculate the work done to move a box.

Work is the product of force and distance or displacement.

[tex]W= F*d[/tex]

A 10 Newton force is applied horizontally on the box. Since the surface is frictionless, there is no force of friction, and the net force is 10 Newtons. The force moves the box 10 meters.

F= 10  N d= 10 m

Substitute the values into the formula.

[tex]W= 10 \ N * 10 \ m[/tex]

Multiply.

[tex]W= 100 \ N*m[/tex]

Let's convert the units. 1 Newton meter is equal to 1 Joule, therefore our answer of 100 Newton meters is equal to 100 Joules.

[tex]W= 100 \ J[/tex]

100 Joules of work was done to move the box.

Water pressurized to 3.5 x 105 Pa is flowing at 5.0 m/s in a horizontal pipe which contracts to 1/2 its former radius. a. What are the pressure and velocity of the water after the contraction

Answers

Answer:

Explanation:

Using the Continuity equation

v X A = v' xA'

so if A is 1/2of A' then A velocity must be 2 times the A'

after-contraction v = 2 x 5.0m/s = 10m/s

Using the Bernoulli equation

p₁ + ½ρv₁² + ρgh₁ = p₂ + ½ρv₂² + ρgh₂

, the "h" terms cancel

3.5 x 10^ 5Pa + ½ x 1000kg/m³x (5.0m/s)² = p₂ + ½ x 1000kg/m³ x (10m/s)²

p₂ = 342500pa

A single slit is illuminated by light of wavelengths λa and λb, chosen so that the first diffraction minimum of the λa component coincides with the second minimum of the λb component. (a) If λb = 350 nm, what is λa? For what order number mb (if any) does a minimum of the λb component coincide with the minimum of the λa component in the order number

Answers

Answer:

λ_A = 700 nm ,   m_B = m_a 2

Explanation:

The expression that describes the diffraction phenomenon is

         a sin θ = m λ

where a is the width of the slit, lam the wavelength and m an integer that writes the order of diffraction

a) They tell us that now lal_ A m = 1

         a sin θ = λ_A

coincidentally_be m = 2

          a sin  θ = m λ_b

as the two match we can match

         λ _A = 2 λ _B

         λ_A = 2 350 nm

         λ_A = 700 nm

b)

For lam_B

       a sin  λ_A  = m_B  λ_B

For lam_A

        a sin θ_A = m_ λ_ A

to match they must have the same angle, so we can equal

           m_B  λ_B = m_A  λ_A

           m_B = m_A  λ_A / λ_B

           m_b = m_a 700/350

           m_B = m_a 2

A person looks horizontally at the edge of a swimming pool. If its length is 5 m, and the pool is filled to the surface, to what depth (in m) could the observer see

Answers

Answer:

The observer could see to a depth of 4.38 m

Explanation:

Please check attachment for diagram.

Mathematically, from Snell law;

n1sin theta = n2 sin theta

1 * sin 90 = n2 * sin θR

where n2 = 1.33

1/1.33 = sin θR

Sin θR = 0.7519

θR = arc sin 0.7519

θR = 48.76

Now to get the height, we use the triangle

Using trigonometric ratio;

Tan( 90- θR) = H/5

H = 5 Tan( 90 - θR)

H = 5 Tan( 90-48.76)

H = 5 Tan41.24

H = 4.38 m

A deep-space vehicle moves away from the Earth with a speed of 0.870c. An astronaut on the vehicle measures a time interval of 3.10 s to rotate her body through 1.00 rev as she floats in the vehicle. What time interval is required for this rotation according to an observer on the Earth

Answers

Answer:

t₀ = 1.55 s

Explanation:

According to Einstein's Theory of Relativity, when an object moves with a speed comparable to speed of light, the time interval measured for the event, by an observer in  motion relative to the event is not the same as measured by an observer at rest.

It is given as:

t = t₀/[√(1 - v²/c²)]

where,

t = time measured by astronaut in motion = 3.1 s

t₀ = time required according to observer on earth = ?

v = relative velocity = 0.87 c

c = speed of light

3.1 s = t₀/[√(1 - 0.87²c²/c²)]

(3.1 s)(0.5) = t₀

t₀ = 1.55 s

Answer:

The time interval required for this rotation according to an observer on the Earth = [tex]6.29sec[/tex]

Explanation:

Time interval required for this rotation according to an observer on the Earth is given as [tex]\delta t[/tex]

where,

[tex]t_o = 3.1\\\\v = 0.87[/tex]

[tex]\delta t = \frac{t_o}{\sqrt{1-\frac{v^2}{c^2}}}\\\\\delta t = \frac{3.1}{\sqrt{1-(\frac{0.87c}{c})^2}}\\\\\delta t = 6.29sec[/tex]

For more information visit

Air is compressed polytropically from 150 kPa, 5 meter cube to 800 kPa. The polytropic exponent for the process is 1.28. Determine the work per unit mass of air required for the process in kilojoules
a) 1184
b) -1184
c) 678
d) -678

Answers

Answer:

wegkwe fhkrbhefdb

Explanation:B

the rate of cooling determines ....... and ......​

Answers

Answer:

freezing point and melting point

Find an analytic expression for p(V)p(V)p(V), the pressure as a function of volume, during the adiabatic expansion.

Answers

Answer:

In an adiabatic process we have

pV γ = const..

This explains that the pressure is a function of volume, p ( V ) ,

So can be written as:

p ( V ) × V γ = p 0 V γ 0 ,

or p ( V ) = p 0 V 0 / V γ

= p 0 V 0 / V ^(7 / 5)

Consider a hydraulic lift that uses an input piston with an area of 0.5m2. An input force of 15N is exerted on this piston. If the output piston has an area of 3.5m? What is the output force?

Answers

Answer:

The output force of the piston is 105 N.

Explanation:

Given;

the area of the input piston, A₁ = 0.5 m²

the input force of the piston, F₁ = 15 N

the area of the output piston, A₀ = 3.5 m²

the output force of the piston, F₀ = ?

The pressure of the  hydraulic lift is given by;

[tex]P = \frac{F}{A}[/tex]

where;

P is the hydraulic pressure

F is the piston force

A is the area of the piston

[tex]P = \frac{F}{A} \\\\\frac{F_o}{A_o} = \frac{F_i}{A_i} \\\\F_o = \frac{F_iA_o}{A_i} \\\\F_o = \frac{15*3.5}{0.5} \\\\F_o = 105 \ N[/tex]

Therefore,  the output force of the piston is 105 N.

What is the separation in meters between two slits for which 594 nm orange light has its first maximum at an angle of 32.8°?

Answers

Answer:

1.1micro meter

Explanation:

Given that

Constructive interference is

ma = alpha x sin theta

Alpha = 1 x 594 x10^ -9/ sin 32.8°

= 1.1 x 10^ -6m

Explanation:

A parallel-plate capacitor consists of two square plates, size L×L, separated by distance d. The plates are given charge ±Q . What is the ratio Ef/Ei of the final electric field strength Ef to the initial electric field strength Ei if:
a. Q is doubled?
b. L is doubled?
c. d is doubled?

Answers

Answer:

Using

A. .E = σ/εo = (q/A)/εo = = q/Aεo so if q = 2q, then

Ef/Ei = 2

B. If L is 2L then Ef = q/4Aεo and

Ef/Ei = 1/4

C. The electric field strength is not effected by d and as long as σ is unchanged, Ef/Ei = 1

A 1.8-mole sample of an ideal gas is allowed to expand at a constant temperature of 250 K. The initial volume is 34 L and the final volume is 80 L. How much work does the gas perform on its container? Let the ideal-gas constant R = 8.314 J/(mol • K).

Answers

Answer:

3201.304 J

Explanation:

Use ideal gas equation to initial stage:

PV=nRT

P * 0.034 = 1.8 * 8.314 * 250

P = 110038.2353 Pa

Use ideal gas equation to final stage:

PV=nRT

P * 0.08 = 1.8 * 8.314 * 250

P = 46766.25 Pa

Process is isothermal (constant temperature )

Therefore,

Work= C ln (V2/V1)

(P1V1=P2V2=C)

(Above equation is taken by integration of P.dv)

Work = P1V1 ln (V2/V1) = P2V2 ln (V2/V1)

By substituting above data to the equation:

Work = (110038.2353 * 0.034) * ln (0.08/0.034)

Work = 3201.304 J

Matter's resistance to a change in motion is called _____ and is directly proportional to the mass of an object

Answers

Answer:

Matter's resistance to a change in motion is called INERTIA and is directly proportional to the mass of an object.

Explanation:

If two firecrackers produce a sound level of 81 dBdB when fired simultaneously at a certain place, what will be the sound level if only one is exploded?

Answers

Answer:

77.96dB

Explanation:

Recall that decibels are a unit of measuring intensity of sound, and depend on the logarithm of the intensity

the intensity, measured in decibels is given by:

I(db)=10log(I/I0)

I is the intensity in MKS units; I0 is the threshold intensity for human hearing (10^-12 W/m^2)

Thus, if the two sounds together have a dB of 81, we know:

81=10log(I/I0)

using the data above, we can find the intensity of the two sounds to be

0.000125 W/m^2

therefore, one firecracker has an intensity half of that, or 0.0000625W/m^2

now use this value to find the dB of one firecracker:

I(dB0=10log(0.0000625/10^-12)=77.96dB

A father and his son want to play on a seesaw. Where on the seesaw should each of them sit to balance the torque?

Answers

Answer:

A The father should sit closer to the pivot.

C The longer wrench makes the job easier because less force is needed when there is more distance from the pivot.

A As far from the head of the hammer as possible because this will maximize torque.

D at the opposite side of the seesaw towards the middle

:) gl

Explanation:

If a father and his son want to play on a seesaw then to balance the torque of the seesaw the father should sit near the pivot as he had more weight as compared to his son, while the son should sit a little farther from the pivot point as compared to his father.

What is the mechanical advantage?

Mechanical advantage is defined as a measure of the ratio of output force to input force in a system, It is used to analyze the forces in simple machines like levers and pulleys.

Mechanical advantage = output force(load) /input force (effort)

As given in the problem statement If a father and his son wish to play on a seesaw,

The father should sit close to the pivot because he weighs more than his son, and the son should sit a little farther away from the pivot point than his father. This will help balance the torque of the seesaw.

Thus, the father should sit near the pivot on the one side and the son should sit a little farther from the pivot of a seesaw on the other side.

Learn more about Mechanical advantages, here

brainly.com/question/16617083

#SPJ2

Which of the following measures is equal to 700 km?

Answers

Answer:

1km=1000m

700km=

700×1000=700000

=700000metres

hope this helps

A body is thrown vertically upwards with a speed of 95m / s and after 7s it reaches its maximum height. How fast does it reach its maximum height? What was the maximum height reached?

Answers

Explanation:

u = 95 m/sec ( Initial speed)

t = 7 sec ( Time of ascent)

According to Equations of Motion :

[tex]s = ut - \frac{1}{2} g {t}^{2} [/tex]

Max. Height = 95 * 7 - 4.9 * 49 = 424. 9 = 425 m

Answer:

332.5 m

Explanation:

At the maximum height, the velocity is 0.

Given:

v₀ = 95 m/s

v = 0 m/s

t = 7 s

Find: Δy

Δy = ½ (v + v₀) t

Δy = ½ (0 m/s + 95 m/s) (7 s)

Δy = 332.5 m

Two separate disks are connected by a belt traveling at 5m/s. Disk 1 has a mass of 10kg and radius of 35cm. Disk 2 has a mass of 3kg and radius of 7cm.
a. What is the angular velocity of disk 1?
b. What is the angular velocity of disk 2?
c. What is the moment of inertia for the two disk system?

Answers

Explanation:

Given that,

Linear speed of both disks is 5 m/s

Mass of disk 1 is 10 kg

Radius of disk 1 is 35 cm or 0.35 m

Mass of disk 2 is 3 kg

Radius of disk 2 is 7 cm or 0.07 m

(a) The angular velocity of disk 1 is :

[tex]v=r_1\omega_1\\\\\omega_1=\dfrac{v}{r_1}\\\\\omega_1=\dfrac{5}{0.35}\\\\\omega_1=14.28\ rad/s[/tex]

(b) The angular velocity of disk 2 is :

[tex]v=r_2\omega_2\\\\\omega_2=\dfrac{v}{r_2}\\\\\omega_2=\dfrac{5}{0.07}\\\\\omega_2=71.42\ rad/s[/tex]

(c) The moment of inertia for the two disk system is given by :

[tex]I=I_1+I_2\\\\I=\dfrac{1}{2}m_1r_1^2+\dfrac{1}{2}m_2r_2^2\\\\I=\dfrac{1}{2}(m_1r_1^2+m_2r_2^2)\\\\I=\dfrac{1}{2}\times (10\times (0.35)^2+3\times (0.07)^2)\\\\I=0.619\ kg-m^2[/tex]

Hence, this is the required solution.

can you guys pls also solve for average speed.

Answers

Answer:

d_t = 3.05km

v_a = 4.3km/h

Explanation:

42mins*(2/3) = 28mins

42mins-28mins = 14mins

d = v*t

d_1 = (4km/h)*(1h/60mins)*(28mins)

d_1 = 1.87km

d_2 = (5km/h)*(1h/60mins)*(14mins)

d_2 = 1.17km

d_t = d_1+d_2

d_t = 1.87km+1.17km

d_t = 3.05km

v_a = (v_1+v_2)/2

v_a = [(2*4km/h)+5km/h)]/3

v_a = 4.3km/h

Consider 1 mol an ideal gas at 28∘ C and 1.06 atm pressure. To get some idea how close these molecules are to each other, on the average, imagine them to be uniformly spaced, with each molecule at the center of a small cube.

A) What is the length of an edge of each cube if adjacent cubes touch but do not overlap?

B) How does this distance compare with the diameter of a typical molecule? The diameter of a typical molecule is about 10-10 m. (in l/dmolecule)

C) How does their separation compare with the spacing of atoms in solids, which typically are about 0.3 nm apart? (in l/lsolid)

Answers

Answer:

A) Length of an edge = 3.38 × 10^(-9) m

B) 34 times the diameter of a molecule.

C) 11 times the atomic spacing in solids.

Explanation:

A) We will use Avogadro's hypothesis to solve this. It states that 1 mole of gas occupies 22.4 L at STP.

We want to find the volume occupied by 1 mole of gas at 1.06 atm pressure and temperature of 28 °C (= 301 K).

Thus, by the ideal gas equation, we have;

V_mole = (1 × 22.4/273) × (301/1.06) = 23.3 L = 0.0233 m³

Now, since from avogadros number, 1 mole of gas contains 6.02 x 10^(23) molecules, then volume occupied by a molecule is given by;

V_molecule = 0.0233/(6.02 × 10^(23)) m³ = 3.87 x 10^(-26) m³

Thus, length of an edge of the cube = ∛(3.87 × 10^(-26)) = 3.38 × 10^(-9) m

B) We are told that The diameter of a typical molecule is about 10^(-10) m.

Thus, the distance is about;

(3.38 × 10^(-9))/(10^(-10)) ≈ 34 times the diameter of a molecule.

C) We are told that the spacing of atoms is typically are about 0.3 nm apart

Thus;

The separation will be about;

(3.38 × 10^(-9))/(0.3 × 10^(-9)) ≈ 11 times the atomic spacing in solids.

A damped oscillator is released from rest with an initial displacement of 10.00 cm. At the end of the first complete oscillation, the displacement reaches 9.05 cm. When 4 more oscillations are completed, what is the displacement reached

Answers

Answer:

The  displacement is  [tex]A_r = 6.071 \ cm[/tex]

Explanation:

From the question we are told that

   The initial displacement is [tex]A_o = 10 \ cm[/tex]

     The displacement at the end of first oscillation is  [tex]A_d = 9.05 \ cm[/tex]

     

Generally the damping constant of this damped oscillator is mathematically represented as  

           [tex]\eta = \frac{A_d}{A_o}[/tex]

substituting values

           [tex]\eta = \frac{9.05}{10}[/tex]

        [tex]\eta = 0.905[/tex]

The displacement after 4 more oscillation is mathematically represented as

       [tex]A_r = \eta^4 * A_d[/tex]

substituting values

      [tex]A_r = (0.905)^4 * (9.05)[/tex]

      [tex]A_r = 6.071 \ cm[/tex]

Answer:

Displacement reached is 6.0708 cm

Explanation:

Formula for damping Constant "C"

[tex]C^n=\frac{A_2}{A_1}[/tex]                  where n=1,2,3,........n

Where:

[tex]A_2[/tex] is the displacement after first oscillation    

[tex]A_1\\[/tex] is the initial Displacement

[tex]A_1=10\ cm\\A_2=9.05\ cm\\[/tex]

In our case, n=1.

[tex]C=\frac{9.05}{10}\\C=0.905[/tex]

After 4 more oscillation, n=4:

[tex]C^4=\frac{A_6}{A_2}[/tex]                                        

Where:

[tex]A_6[/tex] is the final Displacement after 4 more oscillations.

[tex]A_6=(0.905)^4*(9.05)\\A_6=6.0708\ cm[/tex]

Displacement reached is 6.0708 cm

A rock has mass 1.80 kg. When the rock is suspended from the lower end of a string and totally immersed in water, the tension in the string is 10.8 N . What is the smallest density of a liquid in which the rock will float?

Answers

Answer:

The density is  [tex]\rho_z = 2544 \ kg /m^3[/tex]

Explanation:

From the question we are told that

    The mass of the rock is  [tex]m_r = 1.80 \ kg[/tex]

     The  tension on the string is  [tex]T = 10.8 \ N[/tex]

Generally the weight of the rock is  

        [tex]W = m * g[/tex]

=>     [tex]W = 1.80 * 9.8[/tex]

=>   [tex]W = 17.64 \ N[/tex]

Now the upward force(buoyant force) acting on the rock  is mathematically evaluated as  

        [tex]F_f = W - T[/tex]

substituting values

       [tex]F_f = 17.64 - 10.8[/tex]

      [tex]F_f = 6.84 \ N[/tex]

This buoyant force is mathematically represented as

      [tex]F_f = \rho * g * V[/tex]

Here  [tex]\rho[/tex] is the density of water and it value is [tex]\rho = 1000\ kg/m^3[/tex]

 So

         [tex]V = \frac{F_f}{ \rho * g }[/tex]

        [tex]V = \frac{6.84}{ 1000 * 9.8 }[/tex]

        [tex]V = 0.000698 \ m^3[/tex]

Now for this rock to flow the upward force (buoyant force) must be equal to the length

      [tex]F_f = W[/tex]

      [tex]\rho_z * g * V = W[/tex]

Here z is smallest density of a liquid in which the rock will float

=>     [tex]\rho_z = \frac{W}{ g * V}[/tex]

=>   [tex]\rho_z = \frac{17.64}{ 0.000698 * 9.8}[/tex]

=>   [tex]\rho_z = 2544 \ kg /m^3[/tex]

Other Questions
A balloon is floating around outside your window. The temperature outside is 31 C , and the air pressure is 0.700 atm . Your neighbor, who released the balloon, tells you that he filled it with 4.10 moles of gas. What is the volume of gas inside this balloon? Express to THREE signficant figures. Ch. 13 Trojan war How does the morality of the gods compare to that of the mortals? responses should be between 150-750 words. Choose just ONE of the following questions to answer: 2 1/2 + 3 1/4 = ? pls On the grid, draw a rotation of triangle ABC, a translation of triangle ABC, and a reflection of triangle ABC. Describe clearly how each was done. Please I need examples on using 111km in calculating the actual distance on ground when the map distance and scale is given "ai ngng hc hi ngi s gi nua,ai lin tc hc hi ngi s tr mi iu tuyt vi nht trn i l gi cho tm tr mnh mi mi tr trung". Bn ngh sao v cu ni ny? Hy bnh lun v chia s kinh nghim ca ring bn. ASAP!!! PLEASE help me solve this question! No nonsense answers, and solve with full solutions. Samuel takes 5 hours to make a table and a chair. He takes 9 hours to make 3 chairsand a table. How much time does he take to make a table? On January 1, 2021, Kendall Inc. began construction of an automated cattle feeder system. The system was finished and ready for use on September 30, 2022. Expenditures on the project were as follows: January 1, 2021 $ 235,000 September 1, 2021 $ 342,000 December 31, 2021 $ 342,000 March 31, 2022 $ 342,000 September 30, 2022 $ 235,000 Kendall borrowed $764,000 on a construction loan at 7% interest on January 1, 2021. This loan was outstanding throughout the construction period. The company had $4,570,000 in 7% bonds payable outstanding in 2021 and 2022. Average accumulated expenditures for 2021 was: Which statement describes the role of the organism indicated by the blue arrow in the food web?A secondary consumer that obtains its energy from the consumption of plantsA primary producer that recycles organic molecules from sunlightA primary decomposer that recycles organic molecules by breaking them downA primary consumer that obtains its energy from the consumption of plants Predict how do you think may affect the future this United States of America A graphed system of linear inequalities is shown below.image 28bfc30ab63a4ce9803aa27b02433fa0Which of the following ordered pairs is a solution to the system of inequalities? Select ALL that apply.A (4,1)B (3,4)C (0,3)D (0,1)E (5,2) Matthew originally crafted this Gospel for a group of Christians who needed to become more familiar with the Old Testament.a) trueb) false You are going to receive $80 at the end of each year for the next 12 years. If you invest each of those amounts at 12%, then what amount of money will you have at the end of the 12th year Which pronoun would you use if you do not want to get acquainted with someone ?vous tu The law of conservation of energy states that A: energy is always created and destroyed B: energy cannot be created or destroyed C: energy is unable to change forms D: energy should be saved as often as possible If 2x/3x/10=17/10, then x = ? where c is the speed of light and G is the universal gravitational constant. RBH gives the radius of the event horizon of a black hole with mass ????. In other words, it gives the radius to which some amount of mass ???? would need to be compressed in order to form a black hole. The mass of the Sun is about 1.991030 kg. What would be the radius of a black hole with this mass? She saw her husband in thehospital The half-life of Zn-71 is 2.4 minutes. If one had 100.0 g at the beginning, how many grams would be left after 7.2 minutes has elapsed? Report your answer to 1 decimal place.